LSAT and Law School Admissions Forum

Get expert LSAT preparation and law school admissions advice from PowerScore Test Preparation.

 Zierra28
  • Posts: 17
  • Joined: Aug 12, 2015
|
#20625
So I feel I understand this question (strengthen), it's conclusion (CH4 production could be kept in check if cows were given better quality diets), but the answer (A) seems to have little to do with the conclusion, which is what the question asked. I understand milk/meat production were discussed earlier in the stimulus, but I don't see how that's significant when the conclusion was about decreasing CH4 production. Please help! Thanks!!
 Steve Stein
PowerScore Staff
  • PowerScore Staff
  • Posts: 1153
  • Joined: Apr 11, 2011
|
#20629
Hi Zierra,

That's a great question. The author provides that cows would produce less methane if they were given better quality diets, concluding that the cows' methane production could be kept under control with better quality diets.

As you mentioned, the question that follows requires you to strengthen the conclusion. Answer choice (A) provides that better quality diets also lead cows to produce much more milk and meat. How does this help the author's argument? If, with a better diet, the cows were to produce the same amount of milk and meat, and reduce methane production, that's great. But if the cows were to produce even more milk and meat, then the relative methane production (per gallon or pound of milk or meat produced) would decrease even further.

It's like if Texaco says that their high end gasoline can help keep the emissions you create in check; if the gas just creates fewer emissions, that's great. But if a gallon of that gas were to allow you to go twice as far (hypothetically), that would help reduce emissions further, because you would only have to use half as much gas to go the same distance you would have otherwise (essentially, cutting emissions per mile in half).

Tricky question! Please let me know whether this is clear—thanks!

~Steve
 nrpandolfo
  • Posts: 33
  • Joined: Feb 04, 2018
|
#45486
Hi,

Why is C wrong for this? My prephrase was that the cost of putting cows on high quality diets is bearable, or even more simpler, it is possible to put cows on a higher quality diet in the first place.

Thanks
 Daniel Stern
PowerScore Staff
  • PowerScore Staff
  • Posts: 81
  • Joined: Feb 07, 2018
|
#45511
I'm afraid you might have thrown yourself with a pre-phrase that was a bit too specific for this Strengthen question.

Our conclusion is about keeping methane in check if we give cows better diets, so our strengthen answer is going to support the notion that feeding them the better diets actually would reduce their methane output. Answer choice A gets us there because it posits that, in addition to the premise info that cows eating better diets each produce less methane, A tells us that the better diets also yield more meat and milk, meaning we need fewer cows overall to meet demand. This further supports that the total methane output would be reduced by putting the cows on the better diets.

C talks about whether the farmers are willing to switch; whether we can actually achieve the state of feeding all cows, worldwide, the better diets is actually irrelevant to the stimulus's normative conclusion: we would have less methane production if we fed the cows better diets.

I hope that helps you to understand.

When pre-phrasing for strengthen questions, try and think more in terms of what function the correct answer (i.e. reinforce the notion that the better diets reduce methane) will play in the argument rather than pre-phrasing something too specifc (i.e. the new diets have to cost a reasonable amount).

Best,
Dan
 sblack1998
  • Posts: 10
  • Joined: Feb 05, 2020
|
#90053
I was so off on this question. I chose C and diagramed as follows: high quality feed was less expensive ---> most farmers would feed it to their cows. If most farmers feed their cows with high quality feed --->(from the 3rd sentence) Methane is reduced. Does reducing methane not support/strengthen the conclusion of keeping methane in check? Where did my reasoning go wrong?
User avatar
 atierney
PowerScore Staff
  • PowerScore Staff
  • Posts: 215
  • Joined: Jul 06, 2021
|
#90114
Hello,

In considering strengthen questions, the inquiry concerns how to best strengthen the conclusion of the argument.

Here, the conclusion of the argument is that "methane production from cows could be kept in check if cows were given better-quality diets," or in other words assuming the cows were actually given the better-quality diets. Answer choice C provides support for the fact that farmers would actually give this high quality if it were cheaper, or given a certain condition sufficient. But both conditions, once again, are assumed in the argument's conclusion itself; therefore, neither are required for nor aid the conclusion itself. What is needed is some means to explain how, by giving the cows better-quality diets, the methane production of the world's cows in general would be kept in check. And for that, we have to go to the initial sentence within the stimulus, which mentions the fact that the cow is growing at a rate consistent with the demand for meat and milk. Notice that this fact would indicate that even with the better-quality diets, methane production would still be expected to increase over time, given the increase in cows needed to meet this increased demand for meat and milk. Thus, the correct answer choice should be one that addresses this issue; and such an answer choice is found in A, which states that the milk and meat production-per-cow could be increased as well with the increase in the better-quality diets, thereby eliminating the need to increase the cow population as much with the increased demand for milk and meat.

Let me know if you have further questions on this.
 KG!
  • Posts: 69
  • Joined: May 26, 2020
|
#94834
Hmm, I chose E. I thought A was incorrect because if they are producing more meat, I'm thinking more methane is coming from cows if that makes sense? Not the need for fewer cows. It seem like that the reason A is correct is a stretch honestly. I'm not really sure how to prepare my mind to not think of that conclusion except to note that this was a question near the end of the section, so it probably is a harder one.
 Robert Carroll
PowerScore Staff
  • PowerScore Staff
  • Posts: 1787
  • Joined: Dec 06, 2013
|
#94931
KG!,

The stimulus already says that cows produce less methane when they have a higher-quality diet. Answer choice (A) doesn't deny that, and it's simply an unwarranted assumption to think that methane production correlates with output - the only thing the stimulus has asserted is that methane production correlates with the diet input of cows.

I don't see how answer choice (A)'s strengthening of the argument is a stretch. The answer says that cows with better-quality diets will produce more meat and milk. Further, the stimulus already establishes that they will produce less methane. This looks like a win/win - more production and less pollution, all from doing the same thing.

Robert Carroll

Get the most out of your LSAT Prep Plus subscription.

Analyze and track your performance with our Testing and Analytics Package.